SEQUENCES AND SERIES - 4 (Inequalities based on A.M., G.M. and H.M.)

Properties of A.M, G.M & H.M

Let A, G, $\mathrm{H}$ be the arithmetic, geometric and harmonic means of two positive numbers a & (b).

Then, $A=\frac{a+b}{2}, G=\sqrt{a b}, H=\frac{2 a b}{a+b}$

i. A. $\mathrm{H}=\frac{\mathrm{a}+\mathrm{b}}{2} \cdot \frac{2 \mathrm{ab}}{\mathrm{a}+\mathrm{b}}=\mathrm{ab}=\mathrm{G}^{2}$

i.e. $\mathrm{G}^{2}=$ A.H

$\mathrm{G}$ is the geometric mean between $\mathrm{A} \& \mathrm{H}$.

Again $A-G=\frac{a+b}{2}-\sqrt{a b}=\frac{(\sqrt{a}-\sqrt{b})^{2}}{2}>0$

$\Rightarrow \mathrm{A}>\mathrm{G}$

Also $\mathrm{G}^{2}=\mathrm{A} . \mathrm{H}$

$ \begin{aligned} & \frac{\mathrm{G}}{\mathrm{H}}=\frac{\mathrm{A}}{\mathrm{G}}>1 \\ & \Rightarrow \frac{\mathrm{G}}{\mathrm{H}}>1 \text { or } \mathrm{G}>\mathrm{H} \end{aligned} $

Combining, $\mathrm{A}>\mathrm{G}>\mathrm{H}$

Note: If the numbers are equal, then $A=G=H$. Thus, $A \geq G \geq H$, equality holds when the numbers are equal.

ii. The equation with $a$ and $b$ as its roots is $\mathrm{x}^{2}-2 \mathrm{Ax}+\mathrm{G}^{2}=0$.

or if $A \& G$ be the A.M and G.M between two positive numbers $a \& b$ then $\frac{a}{b}=\frac{A+\sqrt{A^{2}-G^{2}}}{A-\sqrt{A^{2}-G^{2}}}$

iii. If A, G, H be the A.M, G.M, and H.M between three given numbers, a, b and c, then the equation having $a, b, c$ as its roots is

$\mathrm{x}^{3}-3 A \mathrm{x}^{2}+\frac{3 \mathrm{G}^{3}}{\mathrm{H}} \mathrm{x}-\mathrm{G}^{3}=0$

Proof: $A=\frac{a+b+c}{3} \Rightarrow a+b+c=3 A$

$ G=(a b c)^{1 / 3} \Rightarrow G^{3}=a b c $

$ \begin{aligned} & \frac{1}{\mathrm{H}}=\frac{\frac{1}{\mathrm{a}}+\frac{1}{\mathrm{~b}}+\frac{1}{\mathrm{c}}}{3} \Rightarrow \frac{\mathrm{ab}+\mathrm{bc}+\mathrm{ca}}{3 \mathrm{abc}}=\frac{1}{\mathrm{H}} \\ & \text { or } \mathrm{ab}+\mathrm{bc}+\mathrm{ca}=\frac{3 \mathrm{abc}}{\mathrm{H}}=\frac{3 \mathrm{G}^{3}}{\mathrm{H}} \end{aligned} $

Equation having $\mathrm{a}, \mathrm{b}, \mathrm{c}$ as roots is

$ \begin{aligned} & x^{3}-(a+b+c) x+(a b+b c+c a) x^{2}-a b c=0 \\ & x^{3}-3 A x+\frac{3 G^{3}}{H} x-G^{3}=0 \end{aligned} $

Example : For distinct positive numbers $x, y, z$, prove that $(x+y)(y+z)(z+x)>8 x y z$

Show Answer

Solution: We have $\mathrm{A}>\mathrm{G}$

For positive numbers $\mathrm{x}$ and $\mathrm{y}, \mathrm{x}+\mathrm{y}>2 \sqrt{\mathrm{xy}}$

For positive numbers y and $\mathrm{z}, \mathrm{y}+\mathrm{z}>2 \sqrt{\mathrm{yz}}$

For positive numbers $\mathrm{z}$ and $\mathrm{x}, \mathrm{z}+\mathrm{x}>2 \sqrt{\mathrm{zx}}$

Multiplying,

$ (x+y)(y+z)(z+x)>8 \sqrt{x^{2} y^{2} z^{2}} $

i.e.,

$ (x+y)(y+z)(z+x)>8 x y z $

Solved examples

1. If $a, b, c$ are positive then prove that $((1+a)(1+b)(1+c))^{7}>7^{7} a^{4} b^{4} c^{4}$.

Show Answer

Solution: $(1+a)(1+b)(1+c)=1+a+b+c+a b+b c+c a+a b c$

$ \begin{aligned} & >a+b+c+a b+b c+c a+a b c \\ & \geq 7(a \cdot b \cdot c \cdot a b \cdot b c \cdot c a \cdot a b c)^{1 / 7} \quad(\because A \geq G) \end{aligned} $

i.e. $\quad(1+a)(1+b)(1+c) \geq 1+7\left(a^{4} b^{4} c^{4}\right)^{1 / 7}>7\left(a^{4} b^{4} c^{4}\right)^{1 / 7}$

$ ((1+a)(1+b)(1+c))^{7}>7^{7} a^{4} b^{4} c^{4} $

2. Maximum value of $x y z$ for positive values of $x, y, z$ if $y z+z x+x y=12$ is

(a). $64$

(b). $4^{3 / 2}$

(c). $8$

(d). none of these

Show Answer

Solution: Apply $ A \geq G$ for $y z, z x \& x y$

$ \frac{y z+z x+x y}{3}>\left(x^{2} y^{2} z^{2}\right)^{1 / 3} $

$ \begin{aligned} & \frac{12}{3} \geq(\mathrm{xyz})^{2 / 3} \\ & (\mathrm{xyz}) \leq 4^{3 / 2} \\ & \mathrm{xyz} \leq 8 \end{aligned} $

Answer: c

3. Maximum value of $x^{2} y^{3}$ where $x$ & $y$ lie in $1^{\text {st }}$ quadrant in the line $3 x+4 y=5$.

(a). $\frac{5}{16}$

(b). $\frac{3}{8}$

(c). $\frac{5}{8}$

(d). $\frac{3}{16}$

Show Answer

Solution: $x^{2} y^{3}=x \cdot x \cdot y \cdot y \cdot y$

$3x+4 y=\frac{3 x}{2}+\frac{3 x}{2}+\frac{4 y}{3}+\frac{4 y}{3}+\frac{4 y}{3}$

$\mathrm{A} \geq \mathrm{G}$

$ \frac{\frac{3 x}{2}+\frac{3 x}{2}+\frac{4 y}{3}+\frac{4 y}{3}+\frac{4 y}{3}}{5} \geq\left(\frac{3 x}{2} \cdot \frac{3 x}{2} \cdot \frac{4 y}{3} \cdot \frac{4 y}{3} \cdot \frac{4 y}{3}\right)^{1 / 5} $

$ \frac{5}{5} \geq\left(\frac{16 x^{2} y^{3}}{3}\right)^{\frac{1}{5}} $

$\Rightarrow \quad x^{2} y^{3} \leq \frac{3}{16}$

Answer: d

4. If $\mathrm{a}^{2}+\mathrm{b}^{2}+\mathrm{c}^{2}=1=\mathrm{x}^{2}+\mathrm{y}^{2}+\mathrm{z}^{2}$, then maximum value of $\mathrm{ax}+\mathrm{by}+\mathrm{cz}$ is $(\mathrm{a}, \mathrm{b}, \mathrm{c}, \mathrm{x}, \mathrm{y}, \mathrm{z}$ are positive real numbers)

(a). 4

(b). 3

(c). 2

(d). 1

Show Answer

Solution: $A>G$

$ \frac{\mathrm{a}^{2}+\mathrm{x}^{2}}{2}>\sqrt{\mathrm{a}^{2} \mathrm{x}^{2}} \Rightarrow \quad \mathrm{a}^{2}+\mathrm{x}^{2}>2 \mathrm{ax} $

Similarly

$ \begin{aligned} & \mathrm{b}^{2}+\mathrm{y}^{2}>2 \mathrm{by} \\ & \mathrm{c}^{2}+\mathrm{z}^{2}>2 \mathrm{cz} \end{aligned} $

adding, $\left(a^{2}+b^{2}+c^{2}\right)+\left(x^{2}+y^{2}+z^{2}\right)>2(a x+b y+c z)$

$\Rightarrow \quad \mathrm{ax}+\mathrm{by}+\mathrm{cz}<\frac{1+1}{2}$

Answer: d

5. If $a, b, c$ are positive then the minimum value of $\frac{a}{b+c}+\frac{b}{c+a}+\frac{c}{a+b}$ is

(a). $\frac{2}{3}$

(b). $\frac{3}{2}$

(c). 1

(d). none of these

Show Answer

Solution: Apply $A \geq H$ for $\frac{1}{\mathrm{~b}+\mathrm{c}}, \frac{1}{\mathrm{c}+\mathrm{a}}, \frac{1}{\mathrm{a}+\mathrm{b}}$

$ \begin{aligned} & \frac{\frac{1}{b+c}+\frac{1}{c+a}+\frac{1}{a+b}}{3} \geq \frac{3}{a+b+b+c+c+a} \\ \Rightarrow \quad & \frac{1}{b+c}+\frac{1}{c+a}+\frac{1}{a+b} \geq \frac{9}{2(a+b+c)} \\ \Rightarrow \quad & \frac{a+b+c}{b+c}+\frac{a+b+c}{c+a}+\frac{a+b+c}{a+b} \geq \frac{9}{2} \\ \Rightarrow \quad & \frac{a}{b+c}+\frac{b}{c+a}+\frac{c}{a+b}+3 \geq \frac{9}{2} \\ \Rightarrow \quad & \frac{a}{b+c}+\frac{b}{c+a}+\frac{c}{a+b} \geq \frac{3}{2} \end{aligned} $

Answer: b

6. If $\mathrm{x}, \mathrm{y}, \mathrm{z}$ are three positive numbers, then $(\mathrm{x}+\mathrm{y}+\mathrm{z})\left(\frac{1}{\mathrm{x}}+\frac{1}{\mathrm{y}}+\frac{1}{\mathrm{z}}\right)>………….$

(a). $3$

(b). $9$

(c). $\frac{1}{3}$

(d). none of these

Show Answer

Solution: $\quad \frac{x+y+z}{3}>(x y z)^{1 / 3}$

Also $\frac{\frac{1}{x}+\frac{1}{y}+\frac{1}{z}}{3}>\left(\frac{1}{x} \cdot \frac{1}{y} \cdot \frac{1}{z}\right)^{1 / 3}$ $(\because A>G)$

Multiplying, $\quad \frac{(\mathrm{x}+\mathrm{y}+\mathrm{z})\left(\frac{1}{\mathrm{x}}+\frac{1}{\mathrm{y}}+\frac{1}{\mathrm{z}}\right)}{9}>1$

Answer: b (or apply $\mathrm{A} \geq \mathrm{H}$ for $\mathrm{x}, \mathrm{y}, \mathrm{z}$ to get the result)

7. Prove that: ${ }^{n} C _{1} \cdot\left({ }^{n} C _{2}\right)^{2}\left({ }^{n} C _{3}\right)^{3} \ldots \ldots . .\left({ }^{n} C _{n}\right)^{n} \leq\left(\frac{2^{n}}{n+1}\right)$

Show Answer

Solution: Let $\mathrm{S}={ }^{n} \mathrm{C} _{1}+2^{\mathrm{n}} \mathrm{C} _{2}+3^{\mathrm{n}} \mathrm{C} _{3}+$ n ${ }^{n} C _{n}$

$ =\sum _{r=1}^{n} \quad r \cdot{ }^{n} C _{r}=\sum _{r=1}^{n} n \cdot{ }^{n-1} C _{r-1}=n 2^{n-1} $

Now $A \geq G$

$\frac{{ }^{n} C _{1}+\left({ }^{n} C _{2}+{ }^{n} C _{2}\right)+\left({ }^{n} C _{3}+{ }^{n} C _{3}+{ }^{n} C _{3}\right)+\ldots . .\left({ }^{n} C _{n}+{ }^{n} C _{n}+\ldots \ldots+{ }^{n} C _{n}\right)}{1+2+3+\ldots \ldots \ldots .+n}$

$\geq\left({ }^{n} C _{1}\left({ }^{n} C _{2}\right)^{2} \ldots \ldots . .\left({ }^{n} C _{n}\right)^{n}\right)^{\frac{1}{1+2+\ldots .+n}}$

$\Rightarrow \quad \frac{\mathrm{n} \cdot 2^{\mathrm{n}-1}}{\frac{\mathrm{n}(\mathrm{n}+1)}{2}} \geq\left({ }^{\mathrm{n}} \mathrm{C} _{1}\left({ }^{\mathrm{n}} \mathrm{C} _{2}\right)^{2} \ldots \ldots . .\left({ }^{\mathrm{n}} \mathrm{C} _{\mathrm{n}}\right)^{\mathrm{n}}\right)^{\frac{2}{(\mathrm{n}+1)}}$

$\Rightarrow \quad\left({ }^{n} C _{1} \cdot\left({ }^{n} C _{2}\right)^{2}\left({ }^{n} C _{3}\right)^{3} \ldots \ldots .\left({ }^{n} C _{n}\right)^{n}\right) \leq\left(\frac{2^{n}}{n+1}\right)^{\frac{n(n+1)}{2}}=\left(\frac{2^{n}}{n+1}\right)^{n+1} C _{2}$

8. If $a, b, c, d$ are in H.P. then

(a). $ \mathrm{a}+\mathrm{d}>\mathrm{b}+\mathrm{c}$

(b). $a d>b c$

(c). $a d=b c$

(d). none of these

Show Answer

Solution: $ \mathrm{a}, \mathrm{b}, \mathrm{c}$ are in H.P $\Rightarrow \mathrm{b}$ is the H.M of $\mathrm{a} \& \mathrm{c}$.

A.M of $\mathrm{a} \& \mathrm{c}=\frac{\mathrm{a}+\mathrm{c}}{2}$

we have $\mathrm{A}>\mathrm{H} \Rightarrow \frac{\mathrm{a}+\mathrm{c}}{2}>\mathrm{b} \Rightarrow \mathrm{a}+\mathrm{c}>2 \mathrm{~b}$

Similarly $ b+d>2 c$

Adding, $ \mathrm{a}+\mathrm{b}+\mathrm{c}+\mathrm{d}>2 \mathrm{~b}+2 \mathrm{c} \quad \Rightarrow \quad \mathrm{a}+\mathrm{d}>\mathrm{b}+\mathrm{c}$

Also $a, b, c$ are in H.P. $\Rightarrow b$ is the H.M of $a \& c$

G.M of $\mathrm{a} \& \mathrm{c}=\sqrt{\mathrm{ac}}$

$\mathrm{G}>\mathrm{H} \Rightarrow \sqrt{\mathrm{ac}}>\mathrm{b}$

Similarly $\sqrt{\mathrm{b}} \mathrm{d}>\mathrm{c}$

Multiplying, $\sqrt{\mathrm{abcd}}>\mathrm{bc} \quad \Rightarrow \quad \mathrm{ad}>\mathrm{bc}$

Answer: a, b

Practice questions

1. If $\mathrm{a}+\mathrm{b}+\mathrm{c}=1$, then find $k$ such that

$\frac{\mathrm{k}}{27 \mathrm{abc}}>\left(\frac{1}{\mathrm{a}}-1\right)\left(\frac{1}{\mathrm{~b}}-1\right)\left(\frac{1}{\mathrm{c}}-1\right)>\mathrm{k}$

(a). 8

(b). 7

(c). 3

(d). none of these

Show Answer Answer: (a)

2. A rod of fixed length $k$ slides along the coordinate axes. If it meets the axes at $A(a, 0)$ and $B(0, b)$, then the minimum value of $\left(a+\frac{1}{a}\right)^{2}+\left(b+\frac{1}{b}\right)^{2}$ is

(a). $0$

(b). $8$

(c). $\mathrm{k}^{2}-4+\frac{4}{\mathrm{k}^{2}}$

(d). $\mathrm{k}^{2}+4+\frac{4}{\mathrm{k}^{2}}$

Show Answer Answer: (d)

3. If positive numbers $a, b, c$ be in H.P, the equation $x^{2}-k x+2 b^{101}-a^{101}-c^{101}=0(k \varepsilon R)$ has

(a). both roots imaginary

(b). one root is positive and other is negative

(c). both roots positive

(d). both roots negative

Show Answer Answer: (b)

4. If $\mathrm{n} \varepsilon \mathrm{N}, \mathrm{n}^{\mathrm{n}}\left(\frac{\mathrm{n}+1}{2}\right)^{2 \mathrm{n}}>\mathrm{k}$ where $\mathrm{k}$ is

(a). $(2 n !)^{3}$

(b). $2(n !)^{3}$

(c). $(n !)^{3}$

(d). none of these

Show Answer Answer: (c)

5. If $x, y, z \in R^{+}$, then is $\frac{y z}{y+z}+\frac{x z}{x+z}+\frac{x y}{x+y}$ is always

(a). $\leq \frac{1}{2}(x+y+z)$

(b). $\geq \frac{1}{3} \sqrt{\mathrm{xyz}}$

(c). $\leq \frac{1}{3}(\mathrm{x}+\mathrm{y}+\mathrm{z})$

(d). $\geq \frac{1}{2} \sqrt{\mathrm{xyz}}$

Show Answer Answer: (a)

6. $\sum _{\mathrm{i}=0}^{\infty} \sum _{\mathrm{j}=0}^{\infty} \sum _{\mathrm{k}=0}^{\infty} \frac{1}{3^{\mathrm{i}} 3^{\mathrm{j}} 3^{\mathrm{k}}}$ is

$(\mathrm{i} \neq \mathrm{j} \neq \mathrm{k})$

(a). $\frac{1}{27}$

(b). $\frac{81}{208}$

(c). $1$

(d). none of these

Show Answer Answer: (b)

7. Minimum value of $\frac{(\mathrm{x}-1)(\mathrm{x}-2)}{\mathrm{x}-3}: \forall \mathrm{x}>3$ is

(a). $3 \sqrt{2}-2$

(b). $3+2 \sqrt{2}$

(c). $3+2 \sqrt{3}$

(d). $3 \sqrt{2}+2$

Show Answer Answer: (b)

8. The least value of $6 \tan ^{2} \theta+54 \cot ^{2} \theta+18$ is

i. 54 when A.M. $\geq$ G.M is applied for $6 \tan ^{2} \theta, 54 \cot ^{2} \theta, 18$

ii. 54 when $\mathrm{A} . \mathrm{M} \geq \mathrm{G} . \mathrm{M}$ is applied for $6 \tan ^{2} \theta, 54 \cot ^{2} \theta$ and 18 is added further.

iii. 78 when $\tan ^{2} \theta=\cot ^{2} \theta$.

(a). (iii) is correct

(b). (i) is correct (ii) is flase

(c). (i) and (ii) are correct

(d). none of these

Show Answer Answer: (c)

9. If A, G, H are A.M, G.M, H.M between the same two numbers, such that $\mathrm{A}-\mathrm{G}=15$ and $\mathrm{A}-\mathrm{H}=27$, then the numbers are

(a). 100, 50

(b). 120, 30

(c). 90, 60

(d). none of these

Show Answer Answer: (b)

10. If $a, b, c \varepsilon R$, the square root of $a^{2}+b^{2}+c^{2}-a b-b c-a c$ is greater than or equal to

(a). $\frac{\sqrt{3}}{2} \max \{|\mathbf{b}-\mathrm{c}|,|\mathrm{c}-\mathrm{a}|,|\mathrm{a}-\mathrm{b}|\}$

(b). $\frac{3}{2} \max \{|\mathrm{b}-\mathrm{c}|,|\mathrm{c}-\mathrm{a}|,|\mathrm{a}-\mathrm{b}|\}$

(c). $\max \{|\mathrm{b}-\mathrm{c}|,|\mathrm{c}-\mathrm{a}|,|\mathrm{a}-\mathrm{b}|\}$

(d). $\frac{\sqrt{3}}{4} \max \{|\mathbf{b}-\mathrm{c}|,|\mathrm{c}-\mathrm{a}|,|\mathrm{a}-\mathrm{b}|\}$

Show Answer Answer: (a)

11. If $\mathrm{x} _{1}, \mathrm{x} _{2}, \mathrm{x} _{3}, \mathrm{x} _{4}$ are four positive real numbers such that $\mathrm{x} _{1}+\frac{1}{\mathrm{x} _{2}}=4, \mathrm{x} _{2}+\frac{1}{\mathrm{x} _{3}}=1, \mathrm{x} _{3}+\frac{1}{\mathrm{x} _{4}}=4, \mathrm{x} _{4}+\frac{1}{\mathrm{x} _{1}}=1$ then

(a). $\mathrm{x} _{1}=\mathrm{x} _{3}$ and $\mathrm{x} _{2}=\mathrm{x} _{4}$

(b). $\mathrm{x} _{2}=\mathrm{x} _{4}$ but $\mathrm{x} _{1} \neq \mathrm{x} _{3}$

(c). $\mathrm{x} _{1} \mathrm{x} _{2}=1, \mathrm{x} _{3} \mathrm{x} _{4}=-1$

(d). $\mathrm{x} _{3} \mathrm{x} _{4}=1, \mathrm{x}, \mathrm{x} _{2} \neq 1$

Show Answer Answer: (a)

12. If $\mathrm{a}, \mathrm{b}, \mathrm{c}>0$ and $\mathrm{a}(1-\mathrm{b})>\frac{1}{4}, \mathrm{~b}(1-\mathrm{c})>\frac{1}{4}, \mathrm{c}(1-\mathrm{a})>\frac{1}{4}$, then

(a). never possible

(b). always true

(c). cannot be discussed

(d). none of these

Show Answer Answer: (a)

13. If $\mathrm{a}, \mathrm{b}, \mathrm{c}$ are the sides of a triangle, then $\frac{1}{\mathrm{~b}+\mathrm{c}}, \frac{1}{\mathrm{c}+\mathrm{a}}, \frac{1}{\mathrm{a}+\mathrm{b}}$ are also the sides of the triangle is

(a). sometimes true

(b). always true

(c). cannot be discussed

(d). never true

Show Answer Answer: (b)

14. Given $\mathrm{n}^{4}<10^{\mathrm{n}}$ for a fixed positive integer $\mathrm{n} \geq 2$, then

(a). $(\mathrm{n}+1)^{4}<10^{\mathrm{n}+1}$

(b). $(\mathrm{n}+1)^{4}>10^{\mathrm{n}+1}$

(c). nothing can be said

(d). none of these

Show Answer Answer: (a)


sathee Ask SATHEE

Welcome to SATHEE !
Select from 'Menu' to explore our services, or ask SATHEE to get started. Let's embark on this journey of growth together! ЁЯМРЁЯУЪЁЯЪАЁЯОУ

I'm relatively new and can sometimes make mistakes.
If you notice any error, such as an incorrect solution, please use the thumbs down icon to aid my learning.
To begin your journey now, click on

Please select your preferred language
рдХреГрдкрдпрд╛ рдЕрдкрдиреА рдкрд╕рдВрджреАрджрд╛ рднрд╛рд╖рд╛ рдЪреБрдиреЗрдВ